6 More about primes (non-examinable)

Bertrand postulated in 1845 that for every \(n \in \mathbb{N}\) there is always a prime between \(n\) and \(2n\) (\(n \leq p < 2n\)).
The primes \(2, 5, 11, 23, 47, 89, 179, 359, 719, 1439, 2879\) show it to be true for \(n \leq 2^{11}\). Bertrand checked it for \(n < 3\,000\,000\). Chebychev (1850) gave a proof. Erds (1932) gave an elementary proof based on the properties of \(\binom{2n}{n}\).

Lemma 6.1 \[\begin{align*} \binom{2n}{n} \geq \frac{2^{2n}}{2n + 1}. \end{align*}\]

Proof. Since \(\binom{n}{k + 1} / \binom{n}{k} = \frac{n - k}{k + 1}\), it is evident that \(\binom{n}{k}\) increases for \(k < \frac{n}{2}\), and decreases for \(k > \frac{n}{2}\). In particular, \(\binom{2n}{n} \geq \frac{2^{2n}}{2n + 1}\), the maximum element is at least as big as the average (\(2^{2n}\) is the sum and we have \(2n + 1\) elements).

Lemma 6.2 If \(p \leq n\) is a prime dividing \(\binom{2n}{n}\), then \(p \leq \frac{2n}{3}\).

Proof. Suppose \(\frac{2n}{3} < p \leq n\), then \(p \leq n < \frac{3}{2}p < 2p \leq 2n < 3p\), so the numerator and denominator of \[\begin{align*} \frac{2n (2n - 1) \dots (n + 1)}{n (n - 1) \dots 3 \cdot 2 \cdot 1} \end{align*}\] are divisible by exactly one copy of \(p\). ⨳ as it can then not divide it.

Lemma 6.3 If \(p\) is a prime and \(p^k \mid \binom{2n}{n}\), then \(p^k \leq 2n\).

Proof. The greatest power of \(p\) dividing \(n! = n (n-1) \dots 3 \cdot 2 \cdot 1\). \(\left \lfloor \frac{n}{p} \right \rfloor\) is the no. of multiples of \(p\) upto \(n\), \(\left \lfloor \frac{n}{p^2} \right \rfloor\) is the no. of multiples of \(p^2\) upto \(n\). So the greatest power is \(\left \lfloor \frac{n}{p} \right \rfloor +\left \lfloor \frac{n}{p^2} \right \rfloor +\left \lfloor \frac{n}{p^3} \right \rfloor \dots = \sum_{i \geq 1}\left \lfloor \frac{n}{p^i} \right \rfloor\).

Hence, if \(k\) is a power of \(p\) dividing \(\binom{2n}{n} = \frac{2n !}{(n!)^2}\), then \[\begin{align*} k &\leq \sum_{i \geq 1}\left \lfloor \frac{2n}{p^i} \right \rfloor - 2 \sum_{i \geq 1}\left \lfloor \frac{n}{p^i} \right \rfloor\\ &= \sum^l_{i = 1} \left(\left \lfloor \frac{2n}{p^i} \right \rfloor - 2\left \lfloor \frac{n}{p^i} \right \rfloor \right) \text{ where $l$ is the greatest power of $p$ s.t. $p^l \leq 2n$} \\ &\leq \sum_{i = 1}^k 1 \text{ since } \left \lfloor 2x \right \rfloor - 2 \left \lfloor x \right \rfloor \leq 1 \\ &= l \\ \text{so $k \leq l$ and thus $p^k \leq p^l \leq 2n$}. \end{align*}\]

Lemma 6.4 For all \(m \in \mathbb{N}, \underset{p \leq m \\ \text{$p$ prime}}{\prod} p \leq 4^m\).

Proof. By induction on \(m\). True for \(m = 2\). If \(m > 2\) is even, then \[\begin{align*} \prod_{p \leq m} p = \prod_{p \leq m - 1} p \leq 4^{m - 1} < 4^m. \end{align*}\] If \(m = 2k + 1\) is odd, then all primes \(k + 2 \leq p \leq 2k + 1\) divide \(\binom{2k + 1}{k} = \frac{(2k + 1)!}{k! (k + 1)!} = \frac{(2k + 1) 2k \dots (k + 2)}{k (k - 1) \dots 3 \cdot 2 \cdot 1}\) (as they divide the numerator but not denominator). Thus \(\prod_{k + 2 \leq p \leq 2k + 1} p \leq \binom{2k + 1}{k} = \binom{2k + 1}{k + 1} \leq \frac{2^{2k + 1}}{2} = 4^k\). By the inductive hypothesis, \[\begin{align*} \prod_{p \leq m} p = \prod_{p < k + 1} p \cdot \prod_{k + 2 \leq p \leq 2k + 1} p \leq 4^{k + 1} \cdot 4^k = 4^{2k + 1} \end{align*}\]

Theorem 6.1 (Bertrand's Postulate) For all \(n \in \mathbb{N}\) s.t. \(n \geq 2\), \(\exists\) prime \(p\) with \(n \leq p < 2n\).

Proof. Clearly the prime factors of \(\binom{2n}{n}\) are all \(< 2n\). Suppose the theorem fails. Then all prime factors of \(\binom{2n}{n}\) are in fact \(< n\).
By Lemma 6.2, they are all \(< \frac{2}{3} n\).
Consider the prime factorisation of \(\binom{2n}{n}\). By Lemma 6.3, each prime contributes \(\leq 2n\) to the factorisation. Moreover, if \(p > \sqrt{2n}\), then \(p\) contributes at most \(p\) to the factorisation (since \(p^2 > 2\)). Now by Lemma 6.1 and 6.4 \[\begin{align*} \frac{2^{2n}}{2n + 1} \leq \binom{2n}{n} &\leq \Pi_{p \leq \sqrt{2n}} p \Pi_{\sqrt{2n} < p < \frac{2}{3} n} p \\ &\leq (2n)^{\sqrt{2n}} \cdot \Pi_{p < \frac{2}{3} n} p \\ \text{by Lemma \@ref(lem:sfour), } \Pi_{p < \frac{2}{3} n} p &\leq 4^{\frac{2}{3} n} \\ &\leq (2n)^{\sqrt{2n}} \cdot 4^{\frac{2}{3}n} \\ \frac{4^n}{2n + 1} &\leq (2n)^{\sqrt{2n}} \cdot 4^{\frac{2}{3} n}, \end{align*}\] which fails when \(n\) is large.

How large? \[\begin{align*} 4^{\frac{n}{3}} &\leq (2n + 1)(2n)^{\sqrt{2n}} \\ \text{and } (2n + 1) &\leq (2n)^2 \leq (2n)^{\sqrt{2n}/3} \text{ for } n \geq 18 \\ \text{so } 4^\frac{n}{3} &\leq (2n)^{\frac{4}{3} \sqrt{2n}} \\ \text{or } 4^n &\leq (2n)^{4 \sqrt{2n}} \\ \text{with } r &= \sqrt{2n}, \text{ this is} \\ 4^{\frac{r^2}{2}} &\leq r^{8r} \\ \text{or } 4^r &\leq r^{16} \end{align*}\] which fails when \(r \geq 2^6 = 64\). So proof holds when \(n \geq 2^{11}\), and also true for smaller values of \(n\) as discussed at the beginning of the lecture.